Show by induction that the determinant is equal to n

In summary, the conversation discusses the proof for the statement that the determinant of the matrix $A_n$ is equal to $n!$. It starts with a base case for $n=1,2,3$ and then moves on to the induction hypothesis for $n\geq 2$. The conversation then explores the Laplace expansion and its application to the bottom row, which leads to the conclusion that the statement holds for $n+1$. After some further discussions and clarification, the conversation concludes with the confirmation that strong induction was indeed necessary for this proof.
  • #1
mathmari
Gold Member
MHB
5,049
7
Hey! :eek:

For $n\in \mathbb{N}$ let $A_n$ be the real $n\times n$-matrix with the elements \begin{equation*}a_{ij}=\begin{cases}i , &\text{ if } i=j-1 \\ 1, & \text{ if } i=j \\ -j, & \text{ if } i=j+1 \\ 0 , & \text{ otherwise } \end{cases}\end{equation*}

For $n=1, 2, 3$ we get the matrices :
\begin{equation*}A_1=\begin{pmatrix}1\end{pmatrix} , \ \ A_2=\begin{pmatrix} 1 & 1 \\ -1 & 1\end{pmatrix}, \ \ A_3=\begin{pmatrix}1 & 1 & 0 \\ -1 & 1 & 2 \\ 0 & -2 & 1 \end{pmatrix}\end{equation*}

The determinants are:
\begin{align*}\det (A_1)&=1 \\ \det (A_2)&=\begin{vmatrix} 1 & 1 \\ -1 & 1\end{vmatrix}=1\cdot 1-(-1)\cdot 1=1+1=2 \\ \det (A_3)&=\begin{vmatrix}1 & 1 & 0 \\ -1 & 1 & 2 \\ 0 & -2 & 1 \end{vmatrix}\overset{ \text{ Sarrus } }{ = } \begin{vmatrix}1 & 1 & 0 \\ -1 & 1 & 2 \\ 0 & -2 & 1 \end{vmatrix}\begin{matrix}1 & 1 \\ -1 & 1 \\ 0 & -2\end{matrix} \\ & =1\cdot 1\cdot 1+1\cdot 2\cdot 0+0\cdot (-1)\cdot (-2)-0\cdot 1\cdot 0-(-2)\cdot 2\cdot 1-1\cdot (-1)\cdot 1 \\ & = 1+4+1 =6\end{align*} Now I want to show using induction for $n$ that \begin{equation*}\det (A_n)=n!=n\cdot (n-1)\cdot (n-2)\cdot \ldots \cdot 2\cdot 1\end{equation*}

I have done the following:

Base case: From the above we have that the statement holds for $n=1, 2, 3$, since $\det (A_1)=1=1!$, $\det (A_2)=2=2!$, $\det (A_3)=6=3!$. Induction hypothesis: Let $n\geq 2$. We assume that the statement holds for all $k\leq n$, i.e. that $\det (A_k)=k!$. (IH)

Or do we not use strong induction? (Wondering) Induction step: We want to show that the statement holds then also for $n+1$, i.e. that $\det (A_{n+1})=(n+1)!$.

Could you give me a hint how we could show that? Do we have to expand somehow to use the induction hypothesis? (Wondering)
 
Physics news on Phys.org
  • #2
Hey mathmari! (Wave)

Did you consider the Laplace expansion applied to, say, the bottom most row?
 
  • #3
I like Serena said:
Did you consider the Laplace expansion applied to, say, the bottom most row?
At that row all elements are equal to $0$ except the last two, which are $-(n-1)$ and $1$ respectively, right? (Wondering)
 
  • #4
mathmari said:
At that row all elements are equal to $0$ except the last two, which are $-(n-1)$ and $1$ respectively, right?

For an $(n+1)\times(n+1)$ matrix that should be $-n$ and $1$, shouldn't it? (Wondering)
 
  • #5
I like Serena said:
For an $(n+1)\times(n+1)$ matrix that should be $-n$ and $1$, shouldn't it? (Wondering)

Oh yes, I thought of a $n\times n$-matrix (Blush) But how can we know the formula for the expansion having a general matrix? I got stuck right now. (Wondering)
 
  • #6
mathmari said:
But how can we know the formula for the expansion having a general matrix? I got stuck right now.

Isn't the expansion:
$$\det A_{n+1} = 1\cdot \det A_n - (-n)\cdot \det B_n$$
where $B_n$ is $A_{n+1}$ with column $n$ and the bottom row removed? (Wondering)
 
  • #7
I like Serena said:
Isn't the expansion:
$$\det A_{n+1} = 1\cdot \det A_n - (-n)\cdot \det B_n$$
where $B_n$ is $A_{n+1}$ with column $n$ and the bottom row removed? (Wondering)

We have that \begin{equation*}A_{n+1}=\begin{pmatrix}A_n & v \\ -v^T & 1 \end{pmatrix}\end{equation*} where $v=\begin{pmatrix}0 \\ \vdots \\ 0 \\ n\end{pmatrix}$.

Applying the Laplace expansion to the bottom most row, we get \begin{align*}\det (A_{n+1})&=(-1)^{n+1+n}\cdot (-n)\cdot \det (B_n)+(-1)^{n+1+n+1}\cdot 1\cdot \det (A_n)\\ & =(-1)^{2n+1}\cdot (-n)\cdot \det (B_n)+(-1)^{2n+2}\cdot 1\cdot \det (A_n) \\ & = (-1)\cdot (-n)\cdot \det (B_n)+1\cdot 1\cdot \det (A_n) \\ & = n\cdot \det (B_n)+\det (A_n)\end{align*}

The matrix $B_n$ is the matrix $A_{n-1}$ together with the last column $v$ and the last row the last row of $A_n$, or not? (Wondering)
 
  • #8
mathmari said:
We have that \begin{equation*}A_{n+1}=\begin{pmatrix}A_n & v \\ -v^T & 1 \end{pmatrix}\end{equation*} where $v=\begin{pmatrix}0 \\ \vdots \\ 0 \\ n\end{pmatrix}$.

Applying the Laplace expansion to the bottom most row, we get \begin{align*}\det (A_{n+1})&=(-1)^{n+1+n}\cdot (-n)\cdot \det (B_n)+(-1)^{n+1+n+1}\cdot 1\cdot \det (A_n)\\ & =(-1)^{2n+1}\cdot (-n)\cdot \det (B_n)+(-1)^{2n+2}\cdot 1\cdot \det (A_n) \\ & = (-1)\cdot (-n)\cdot \det (B_n)+1\cdot 1\cdot \det (A_n) \\ & = n\cdot \det (B_n)+\det (A_n)\end{align*}

The matrix $B_n$ is the matrix $A_{n-1}$ together with the last column $v$ and the last row the last row of $A_n$, or not?
(Nod)
 
  • #9
I like Serena said:
(Nod)

If we apply the Laplace expansion to the last column of $B_n$, i.e. at $v$ we get $\det (B_n)=(-1)^{n+n}\cdot n\cdot \det (A_{n-1})=n\cdot \det (A_{n-1})$, or not?

So we get \begin{align*}\det (A_{n+1})&=n\cdot n\cdot \det (A_{n-1})+\det (A_n)\ \overset{\text{ Inductive hypothesis }}{ = } \ n\cdot n\cdot (n-1)!+n!=n\cdot n!+n!=(n+1)\cdot n! \\ & =(n+1)!\end{align*}

(Wondering)
 
  • #10
mathmari said:
If we apply the Laplace expansion to the last column of $B_n$, i.e. at $v$ we get $\det (B_n)=(-1)^{n+n}\cdot n\cdot \det (A_{n-1})=n\cdot \det (A_{n-1})$, or not?

So we get \begin{align*}\det (A_{n+1})&=n\cdot n\cdot \det (A_{n-1})+\det (A_n)\ \overset{\text{ Inductive hypothesis }}{ = } \ n\cdot n\cdot (n-1)!+n!=n\cdot n!+n!=(n+1)\cdot n! \\ & =(n+1)!\end{align*}

Yep.
And it turned out that we indeed needed strong induction. (Nod)
 
  • #11
I like Serena said:
Yep.
And it turned out that we indeed needed strong induction. (Nod)

Great! Thank you! (Smile)
 

1. How does induction prove that the determinant is equal to n?

Induction is a mathematical proof technique that uses the principle of mathematical induction to prove a statement for all natural numbers. In this case, induction starts with a base case and then uses a general formula to prove that the statement holds for all natural numbers, including n. This can be applied to show that the determinant of an n x n matrix is equal to n.

2. Can induction be used to prove other properties of determinants?

Yes, induction can be used to prove various properties of determinants, such as the fact that the determinant is equal to the product of its eigenvalues or that the determinant of a triangular matrix is equal to the product of its diagonal elements.

3. How is the base case chosen in an induction proof for determinants?

The base case for an induction proof for determinants is usually chosen as a 1 x 1 matrix, where the determinant is simply equal to the element in the matrix. This serves as the starting point for the induction proof, and then the general formula can be applied to prove the statement for larger matrices.

4. What is the general formula used in an induction proof for determinants?

The general formula used in an induction proof for determinants is that for an n x n matrix, the determinant can be calculated by expanding along any row or column and then recursively applying the formula for the determinant of an (n-1) x (n-1) matrix. This leads to the base case of a 1 x 1 matrix, as mentioned in the previous question.

5. Can induction be used to prove the converse statement, that if the determinant is equal to n, then the matrix must be an n x n matrix?

Yes, induction can also be used to prove the converse statement. Starting with the base case of a 1 x 1 matrix, we can show that if the determinant is equal to n, then the matrix must be an n x n matrix. This follows from the general formula used in induction proofs for determinants, as mentioned in the previous question.

Similar threads

  • Linear and Abstract Algebra
Replies
4
Views
984
  • Linear and Abstract Algebra
Replies
15
Views
969
  • Linear and Abstract Algebra
Replies
3
Views
1K
  • Linear and Abstract Algebra
2
Replies
52
Views
2K
  • Linear and Abstract Algebra
Replies
2
Views
3K
  • Linear and Abstract Algebra
Replies
2
Views
1K
  • Linear and Abstract Algebra
Replies
8
Views
1K
  • Linear and Abstract Algebra
Replies
1
Views
911
  • Linear and Abstract Algebra
Replies
10
Views
984
  • Linear and Abstract Algebra
Replies
12
Views
2K
Back
Top